Questions tagged [arithmetic-progression]

An arithmetic progression is a (possibly infinite) sequence of numbers such that the difference between consecutive terms is always the same value.

Filter by
Sorted by
Tagged with
15 votes
1 answer
1k views

Arithmetic progressions in stopping time of Collatz sequences

Inspired by the question here, we did a few more simulations of numbers of some specific forms and noticed a pattern. We consider the original $3n+1$ transform where we divide by $2$ if it's even and ...
Yuzuriha Inori's user avatar
2 votes
1 answer
256 views

Explanation about arithmetico-geometric progression (AGP) [closed]

So I came across a formula that looks like: $x_n = \alpha x_{n-1} + \beta$ Since I don't have a strong mathematical background I didn't recognize it was an AGP and as I tried to express $x_n$ with ...
Inspi's user avatar
  • 123
13 votes
2 answers
633 views

A reformulation of Erdős conjecture on arithmetic progressions

Erdős conjecture on arithmetic progressions states that if $S$ is a set of positive integers such that $c(S):=\sum_{n \in S} \frac{1}{n} = \infty$ (large set), then $ \forall \ell \ge 3$ the set $S$ ...
Sebastien Palcoux's user avatar
12 votes
2 answers
720 views

Smallest set such that all arithmetic progression will always contain at least a number in a set

Let $S= \left\{ 1,2,3,...,100 \right\}$ be a set of positive integers from $1$ to $100$. Let $P$ be a subset of $S$ such that any arithmetic progression of length 10 consisting of numbers in $S$ will ...
color's user avatar
  • 169
9 votes
1 answer
973 views

Sums of two squares in arithmetic progressions

Let $r(n)$ denote the number of representations of $n$ as the sum of two squares. Are there any known results on $$\sum _{n\leq x\atop {n\equiv a(q)}}r(n)$$ and in particular is there an asymptotic ...
caws's user avatar
  • 143
3 votes
1 answer
200 views

Gowers norms and three-term arithmetic progressions in the mean

Let $f:\mathbb{Z}^+\to \mathbb{C}$ be bounded. Say we are interested in studying how $f$ behaves in short three-term arithmetic progressions. It is very well-known that we can bound $$\sum_{h\leq H} \...
Nell's user avatar
  • 535
1 vote
0 answers
67 views

Wieferich primes and arithmetic prgressions

Let $p$ be an odd prime number. Let $K$ be a number field with Galois group $G$ and $H$ be a subgroup of $G$ stable under conjugation. Then the Cebotarev density theorem gives that $$\mathcal{L}=\{\...
Zakariae.B's user avatar
3 votes
1 answer
400 views

Covering integers by finitely many arithmetic progressions structure

Assume the positive integers $\mathbb{N}$ are partitioned as $$\mathbb{N} = \cup_{i = 1}^n (a_i + b_i \mathbb{N})$$ where $a_i, b_i \in \mathbb{N}$. Prove that all such partitions are obtained by the ...
Ceka's user avatar
  • 501
5 votes
0 answers
173 views

Large finite subsets of Euclidean space with no isosceles (or approximately isosceles) triangles

Here's a question in combinatorial geometry which feels very much like other questions I'm familiar with but which I can't see how to get a hold of. I'll actually propose two different questions on ...
JSE's user avatar
  • 19.1k
2 votes
1 answer
427 views

Partitioning the positive integers into finitely many arithmetic progressions

From Bóna's A Walk through Combinatorics: Prove or disprove that if we partition the positive integers into finitely many arithmetic progressions then there will be at least one arithmetic ...
VRS's user avatar
  • 39
3 votes
2 answers
390 views

Infinitely many primes in particular progressions

I'm faced with the following problem on primes. Does someone have any clue? Is it (a reformulation of) an open problem? Let $d$ be a positive integer, $d\geq 2$. By Dirichlet's theorem, there is an ...
E. Delaygue's user avatar
2 votes
1 answer
295 views

Homogeneous van der Waerden

The Erdős Discrepancy Problem is whether in any two-coloring of the naturals for any $C$ there is a sequence $d, 2d, \ldots nd$ such that the difference of red and blue numbers in it is more than $C$. ...
domotorp's user avatar
  • 18.4k
2 votes
1 answer
286 views

Extension of Dirichlet's Arithmetic Progression Theorem

Dirichlet's Arithmetic Progression Theorem states that: Given $a, b\in\mathbb{Z^+}$ with $(a,b)=1$, then $a+kb$ is prime for an infinite number of $k\in\mathbb{Z^+}.$ For any given $a$ and $b$ let ...
JMP's user avatar
  • 1,226
1 vote
0 answers
148 views

Arithmetic progression of rationals

We know that the set of rational numbers is countable. For which $n$ can we order all rational numbers as $a_1,a_2,\dots$ so that every subsequence of length $n$ is not an arithmetic progression? For ...
pi66's user avatar
  • 1,199
8 votes
2 answers
3k views

Product of arithmetic progressions

Let $(a_1,a_2\ldots,a_n)$ and $(b_1,b_2,\ldots,b_n)$ be two permutations of arithmetic progressions of natural numbers. For which $n$ is it possible that $(a_1b_1,a_2b_2,\dots,a_nb_n)$ is an ...
pi66's user avatar
  • 1,199
2 votes
0 answers
67 views

Discrepancy related independent vector from tensor product?

Here discrepancy is from $(2.4)$ in https://www.ricam.oeaw.ac.at/files/people/siambook_nied.pdf given by 'The discrepancy $D_N(P) = D_N(x_l,\dots,X_N)$ of the point set $P$ of $N$ points in $\mathbb Z^...
Turbo's user avatar
  • 13.7k
1 vote
0 answers
53 views

Discrepancy bound of integer tensor product sequence?

Here discrepancy is from $(2.4)$ in https://www.ricam.oeaw.ac.at/files/people/siambook_nied.pdf given by 'The (extreme) discrepancy $D_N(P) = D_N(x_l,\dots,X_N)$ of the point set $P$ of $N$ points in $...
Turbo's user avatar
  • 13.7k
5 votes
1 answer
410 views

Does every prime $p$ appear in a $p$-term arithmetic progression of primes? [duplicate]

This is a follow-up to an earlier question. The answer to that question was found on this page. The discussion on OEIS seems to suggest that, for any prime $p$, there should exist a $p$-length ...
Kim's user avatar
  • 4,124
29 votes
4 answers
3k views

Is there an 11-term arithmetic progression of primes beginning with 11?

i.e. does there exist an integer $C > 0$ such that $11, 11 + C, ..., 11 + 10C$ are all prime?
Kim's user avatar
  • 4,124
9 votes
1 answer
316 views

A weak form of the Erdős-Turán conjecture

This question is motivated by the answer of Gowers to the question Erdos Conjecture on arithmetic progressions. Question. (1)-Suppose $A \subset \mathbb{N}$ is such that Lim$_n$ $log(n) \cdot |A \...
Mohammad Golshani's user avatar
1 vote
0 answers
58 views

Catch simple arithmetic progression with spiral bijection [closed]

Consider the simple arithmetic progression ($s, z \in \mathbb{Z}$): $a_1 = s$ $a_{n+1} = a_n + z = s + n\cdot z$ Can somebody devise a procedure (another progression) $b_n$ so that there exists a $...
David Vielhuber's user avatar
2 votes
0 answers
70 views

Closed set containing infinite arithmetic progressions of ANY gap

Let $A\subseteq [0,\infty)$ be a set containing infinite arithmetic progressions of ANY gap, that is, for any $d>0$, there is $t>0$ such that $t+kd\in A$ for all $k\in \mathbb N$. Molter and ...
Thomas Yang's user avatar
6 votes
0 answers
245 views

Is a stronger version of the Erdős-Turan conjecture on arithmetic progessions reasonable? (And related questions.)

Define the size, possibly $\infty$, of a set $S\subseteq \mathbb{N}$ as $|S|=\sum\limits_{n\in S} \frac{1}{n}$. Then the Erdős-Turan conjecture states that if $|S|=\infty$, S must contain arbitrarily ...
Yaakov Baruch's user avatar
41 votes
3 answers
4k views

A game on integers

$A$ and $B$ take turns to pick integers: $A$ picks one integer and then $B$ picks $k > 1$ integers ($k$ being fixed). A player cannot pick a number that his opponent has picked. If $A$ has $5$ ...
Haoran Chen's user avatar
1 vote
0 answers
96 views

large arithmetic progression modulo p (II)

Is it possible to construct a $B$ $\subseteq$ $Z_p(=Z/pZ)$ of cardinal $cp^{\frac{1}{3}}$, for some constant $c$, such that there exists an arithmetic progression of size $c_1p^{\frac{2}{3}}$, for ...
Eshita Mazumdar's user avatar
1 vote
1 answer
229 views

Generalized notion of divisor function?

Divisor function $d(n,m)$ counts the number of $q\in\Bbb N$ with $b<q<m$ such that $n\bmod q\equiv0$. Given $b>0$ what is the correct asymptotic, probabilistic and average case behavior of ...
Turbo's user avatar
  • 13.7k
1 vote
1 answer
221 views

Tighter upper bound for $\sum_{i=1}^kA_i\log(\frac{A_i}{e})$

What is the tightest upper bound one can obtain for the following expression $$\sum_{i=1}^kA_i\log(\frac{A_i}{e})$$ subject to $\sum_{i = 1}^k A_i = C$ in terms of $C$ and $k$? A very loose upper ...
user109523's user avatar
1 vote
0 answers
171 views

Write {1,...,3n} as the disjoint union of arithmetic progressions of length 3 and steps 1, 2,...,n

For $n \equiv 0, 1, 2 \pmod 9$, write $\{\,1,\dots,3n\,\}$ as the disjoint union of arithmetic progressions $A_1, A_2,\dots,A_n$ of length 3, where $A_i$ has step $i$.
Willliam D. Weakley's user avatar
3 votes
0 answers
64 views

What's known about $X$ when $|X(n) + X(n)| < kn$, $n \in \mathbb{N}$, absolute constant $k$?

Let $X$ be an infinite sequence of integers$$x_1 < x_2 < x_3 < \ldots,$$and let $X(n)$ be the set$$\{x_1, x_2, \ldots, x_n\}.$$ Question. What is known about $X$ when we have$$|X(n) + X(n)| &...
user106208's user avatar
2 votes
1 answer
421 views

Essential clarifications on application of pigeonhole principle

In here Lemma $4$ using pigeonhole says: For $T_1,\dots,T_s\in\Bbb R$ with $1\leq T_1,\dots,T_s<p$ and $\prod_{i=1}^sT_i > p^{s−1}$ and any integers $a_1,\dots,a_s$ there is an integer $t$ ...
user avatar
-2 votes
1 answer
200 views

Solutions to a diophantine system

What is the smallest $\gamma_1,\gamma_2,\gamma_3>0$ such that given coprime $p,q=\Theta(\ell)$ and integer $t\geq3$ there are coprime $m,n=\Theta(\ell^{t-1})$ with $(mn,pq)=1$, $\alpha_i\in\Bbb Z$ ...
Turbo's user avatar
  • 13.7k
26 votes
1 answer
1k views

What is the status on this conjecture on arithmetic progressions of primes?

The Green-Tao theorem states that for every $n$, there is an arithmetic sequence of length $n$ consisting of primes. For primes, $p$, let $P(p)$ be the maximum length of an arithmetic progression of ...
Gorka's user avatar
  • 1,825
4 votes
0 answers
1k views

Rational points on the unit circle

Is anything known about any of the following questions about rational points on the unit circle? By “double point” I mean an element of $2C$, where $C$ is the group of rational points on the unit ...
Robin Houston's user avatar
3 votes
1 answer
261 views

(Extremal) arithmetic combinatorics in non-abelian groups

Roth's Theorem states that any subset $A$ of $\{1, \dots, n\}$ with no solution to the equation $$x + y = 2z,\, (x, y, z) \in A^3,\, x \neq y$$ has size $o(n)$. Similar results hold when dealing with ...
Johnny Cage's user avatar
  • 1,543
2 votes
2 answers
381 views

About consecutive integers covered by arithmetic progressions

Help me please to solve the following problem. There are $n$ arithmetic progressions of the form: $$(2i+1)k + x_i,~~~~ i = 1,\ldots,n, k \geq 0$$ Initial integer terms $x_i \geq 0$ are varying. ...
Dmitry Pyatin's user avatar
8 votes
1 answer
389 views

Largeness and arithmetic progression properties of generic reals

Consider the following properties for a subset $A$ of $\mathbb{N}$: (1) $A$ is large: $\sum_{n \in A}$$ 1\over n$$=\infty,$ (2) $A^\infty=\limsup \frac{|A \cap \{ 1, \dots, n\}|}{n} >0$, (3) $A_\...
Mohammad Golshani's user avatar
4 votes
0 answers
526 views

The original proof of Szemerédi's Theorem

Nowadays there are plenty of different proofs of the celebrated Szemerédi's Theorem but for historical reasons I would like to read and understand the original proof. The proof is very tricky and, for ...
Johnny Cage's user avatar
  • 1,543
1 vote
0 answers
116 views

Consecutive integers divisible by consecutive small numbers

Given $n$, what is the largest set of consecutive integers in $[n,2n]$ can we have so that each integer is divisible by a distinct element from $[\log n,2\log n]$ (no partiular order)? So apriori I am ...
Turbo's user avatar
  • 13.7k
8 votes
1 answer
419 views

Primes in arithmetic progression with a moduli equal to a power of 2

I am currently looking for a result stronger than Siegel-Walfisz theorem, which gives an upper bound on the error term $|\pi(x,a,b)-\frac{\pi(x)}{\phi(a)}|$ for particular $a$. The Siegel Walfisz is ...
Serge Boissot's user avatar
8 votes
1 answer
553 views

Subsets of [1..N] with no three-term arithmetic progressions and no large gaps

Let S be a subset of [1..N] containing no three-term arithmetic progression, and let h(S) be the size of the largest gap between two consecutive elements of S. By Roth's theorem, h(S) has to grow ...
JSE's user avatar
  • 19.1k
6 votes
1 answer
407 views

A kind of anti-Ramsey result

In contrast to classic results for arithmetic progressions of arbitrary length in one set at least of any finite partition of $\mathbb N$, it is easy to construct a partition in two sets of integers $...
Feldmann Denis's user avatar
3 votes
1 answer
347 views

Primes in simultaneous arithmetic progressions

Suppose we're given four positive integers $a$, $b$, $c$, $d$ such that $a$ and $b$ are coprime, and $c$ and $d$ are coprime. Is there a non-negative integer $k$ such that both $ak+b$ and $ck+d$ are ...
Filip Nikšić's user avatar
9 votes
1 answer
821 views

Bounded gaps between primes in arithmetic progressions

Has Zhang's work on bounded gaps between primes been extended to the following theorem? For any arithmetic progression $an+b,\gcd(a,b)=1$, there is a constant $H$ (depending only on $a$) such that ...
Wojowu's user avatar
  • 27.4k
5 votes
1 answer
531 views

Siegel-Walfisz for the Möbius function

I am working through the proof of the Bombieri-Vinogradov theorem in Analytic Number Theory (Iwaniec, Kowalski). My problem is that on page 424, it is said that $\mu(m)$ satisfies $D_f(x;q,a)\ll (\...
Mats Hansen's user avatar
6 votes
1 answer
526 views

Adding sets not containing arithmetic progressions of length three by forcing

Consider the following forcing notion: conditions in $\mathbb{P}$ are pairs $(s, N),$ where: 1) $s\in 2^{<\omega}$, 2) $N\in \mathbb{N}$, 3) (by identifying $s$ with a subset of $lh(s)$) $s$ ...
Mohammad Golshani's user avatar
11 votes
2 answers
1k views

Most dense subset of numbers that avoids arbitrarily long arithmetic progressions

The famous Green-Tao theorem says that there exist arbitrarily long sequences of primes in arithmetic progression. I am wondering: How dense can a subset $S \subset \mathbb{N}$ be and still avoid ...
Joseph O'Rourke's user avatar
6 votes
0 answers
375 views

Large sets not containing arithmetic progressions of length 3 in intervals

Given a large enough natural number $N$, let $\Delta_N=\{A \subseteq [N, 2N]: A$ contains no arithmetic progressions of length $3 \},$ where for natural numbers $N<M$ we have $[N, M]=\{N, N+1, ..., ...
Mohammad Golshani's user avatar
4 votes
1 answer
326 views

Thin sets that are well-distributed over arithmetic progressions?

The primes do a nice job of intersecting an arithmetic progression $\{a+dn\}_{n=0}^\infty$ when $a$ and $d$ are coprime (see Dirichlet's theorem). I would like a set of integers $S$ such that the ...
Dustin G. Mixon's user avatar
1 vote
1 answer
367 views

Covering a finite subset of $\mathbb{N}$ with prime arithmetic progressions

Because of a problem I ran into I am trying to get a quick start in covering with arithmetic progressions. First I want to say I am aware of this previously asked question: Covering $\mathbb{N}$ with ...
augu's user avatar
  • 11
4 votes
1 answer
968 views

Smallest prime in an arithmetic progression

Let $\{a_n\}_{n\in\mathbb{N}}$ be defined as $a_n = a + bn$ for some $a, b >0,(a, b) = 1$. Are there good bounds on the minimal $k$ s.t. $a_k$ is prime. It is well known that there are infinitely ...
Mayank Pandey's user avatar